\newfam\bbold \def\bb#1{{\fam\bbold #1}} \font\bbten=msbm10 \font\bbsev=msbm7 \font\bbfiv=msbm5 \textfont\bbold=\bbten \scriptfont\bbold=\bbsev \scriptscriptfont\bbold=\bbfiv \font\bigbf=cmbx12 at 24pt \def\answer{\smallskip{\bf Answer.}\par} \def\endproof{\leavevmode\hskip\parfillskip\vrule height 6pt width 6pt depth 0pt{\parfillskip0pt\medskip}} \let\endanswer\endproof \def\section#1{\vskip18pt plus 6pt minus 6pt\vskip0pt plus 1in\goodbreak\vskip 0pt plus -1in% \noindent{\bf #1}} \let\impl\to \def\nmid{\hskip-3pt\not\hskip2.5pt\mid} \def\problem#1{\bigskip\par\penalty-100\item{#1}} \headline{\vtop{\hbox to \hsize{\strut Math 2106 - Dr. Gupta\hfil Due Thursday 2022-04-26 at 11:59 pm}\hrule height .5pt}} \centerline{\bigbf Homework 12 - Holden Rohrer} \bigskip \noindent{\bf Collaborators:} None \section{Ross Chapter 2, 8.1d, 8.2c, 8.7b} \problem{8.1d} Prove that $\lim {n+6\over n^2-6} = 0.$ \answer Let $\epsilon > 0.$ We will show $N$ such that for all $n>N,$ $|{n+6\over n^2-6}-0| < \epsilon.$ Where $n>6,$ $$|{n+6\over n^2-6}| = {n+6\over n^2-6},$$ and $n^2-6>n^2-36,$ so $${n+6\over n^2-6} < {n+6\over n^2-36} = {1\over n-6}.$$ Choosing $n > N = 6+1/\epsilon,$ and since $n-6 > N-6,$ $${1\over n-6} < {1\over N-6} = {1\over 6+1/\epsilon-6} = {1\over 1/\epsilon} = \epsilon,$$ showing that $\lim {n+6\over n^2-6} = 0.$ \endanswer \problem{8.2c} Find the limit of $c_n = {4n+3\over 7n-5}$ and then prove your claim. \answer $c_n$ converges to $4/7.$ Let $\epsilon > 0.$ For all $$n > N = {41\over 49\epsilon} + {5\over 7},$$ we can show that $|c_n-4/7| < \epsilon.$ $$|c_n-4/7| = \left|{4n+3\over 7n-5}-{4\over 7}\right| = \left|{28n+21 - (28n-20)\over 49n-35}\right| = \left|{41\over 49n-35}\right|,$$ and from $n > N,$ (and where $N\geq 1$) $49n-35 > 49N-35 > 0,$ so $$\left|{41\over 49n-35}\right| > {41\over 49N-35} = {41\over 49({41\over 49\epsilon}+{5\over 7})-35} = {41\over {41\over\epsilon} + 35-35} = {41\epsilon\over 41} = \epsilon.$$ We have now shown that $\lim c_n = 4/7.$ \endanswer \problem{8.7b} Show that $s_n = (-1)^n n$ does not converge. \answer {\bf Disproof.} Assume for the sake of contradiction that $s_n$ converges to $s.$ Let $\epsilon = 1.$ We must have $N$ such that for all $n>N,$ $|s_n-s| < 1$ (implying also $|s_{n+1}-s| < 1$). However, $$|s_n-s_{n+1}| = |s_n-s+s-s_{n+1}| \leq |s_n-s| + |s_{n+1}-s| < 1 + 1 = 2.$$ Yet, $|s_{n+1}-s_n| = |(-1)^n(-(n+1)-n)| = 2n+1,$ and where $n>1,$ we obtain $2n+1>3$ and from the earlier inequality $2n+1<2,$ completing a contradiction. We have then shown that $s_n$ does not converge. \endanswer \bye